1answer.
Ask question
Login Signup
Ask question
All categories
  • English
  • Mathematics
  • Social Studies
  • Business
  • History
  • Health
  • Geography
  • Biology
  • Physics
  • Chemistry
  • Computers and Technology
  • Arts
  • World Languages
  • Spanish
  • French
  • German
  • Advanced Placement (AP)
  • SAT
  • Medicine
  • Law
  • Engineering
Whitepunk [10]
2 years ago
9

The elbow is ____ to the fingers, but ____ to the shoulder

Medicine
2 answers:
In-s [12.5K]2 years ago
6 0
First blank is distal and second is proximal
TEA [102]2 years ago
4 0

Answer:

ik the second blank is proximal

Explanation:

Im not sure about the first

You might be interested in
Write a brief description of the following:
allsm [11]

Answer:

i think its a.b.c.d.e.f.g.h.i.j

Explanation:

because they all have to be in one sentence.

4 0
3 years ago
Heparin 12, 000 units subcutaneous is prescribed. Stock ampoules contain 25, 000 units/ 5ml. What volume in ml should be drawn u
Nutka1998 [239]

Answer: The volume in ml that should be drawn is 2.4ml.

Explanation:

Heparin is an anticoagulant which is administered to prevent the formation of blood

clot in high risk patients. The high risk patients include those undergoing open-heart surgery, bypass surgery, kidney dialysis, and blood transfusions. This medication can be administered subcutaneously ( that is, under the skin) by a health care provider. The exact dosage should be administered to avoid adverse effects.

From the question given,

In 5 ml of stock ampoules, there are 25,000 units.

To calculate how many millilitres will deliver 1 unit, divide by 25,000

5 ÷ 25000 = 0.0002

Therefore 0.0002 ml will deliver 1 Unit.

To calculate the volume that will deliver 12,000 units, multiply by 12,000.

Therefore,

12,000 × 0.0002 = 2.4mL

From the calculation, the volume in ml that should be drawn is 2.4ml

7 0
3 years ago
How can exercise help you live longer?
Anna007 [38]

Answer:

C

Explanation:

I'm pretty sure its C.

4 0
2 years ago
Read 2 more answers
What personal protective equipment (PPE) does a LPN need to wear when changing a dressing for a resident that has MRSA of the wo
Svetlanka [38]

Answer:

Healthcare providers will put on gloves and wear a gown over their clothing while taking care of patients with MRSA. Visitors might also be asked to wear a gown and gloves.

5 0
2 years ago
Which element from the Patients Bill of Right's is the most important? Must state at least 5 reasons behind your answer!
Naily [24]

Answer:

Explan This article will focus on the doctor patient relationship and present areas of greatest concerns. Readers should understand that in most instances, when the word "doctor" is used, the reader may substitute many other names such as nurse, caregiver, hospital, insurer, doctor's office personnel and many others. A patient's rights in relation to their doctors occur at many different levels, and in all specialties. As stated above, the American Medical Association (AMA) outlines fundamental elements of the doctor-patient relationship in their Code of Medical Ethics.  

7 0
3 years ago
Other questions:
  • Which of the following is not part of the mucosa associated lymphatic tissue?
    7·2 answers
  • White blood cells travel through the blood and are a part of the cardiovascular system. Which other system also contains white b
    5·1 answer
  • I need help on question 2!
    7·2 answers
  • Freud believed that women formed weaker superegos, leaving them more vulnerable to psychological disorders.
    13·2 answers
  • List one virus that require that the vaccine is given after exposure.
    14·2 answers
  • The trachea, the bronchial tree, and the lungs are part of which tract?
    15·1 answer
  • A 34-year-old woman presents to the emergency department with a 6-month history of recurrent epistaxis and easy bruising. She ha
    15·1 answer
  • What can a first aid provider do for a person who is having trouble breathing and is trying to use his or her inhaler
    7·1 answer
  • Differentiate between hypertension and<br> hypotension, and list the basic causes of each.
    5·1 answer
  • What type of musculoskeletal disorder can be treated by injections into the trigger points on the body? O multiple sclerosis x c
    13·1 answer
Add answer
Login
Not registered? Fast signup
Signup
Login Signup
Ask question!